Equation of an oscillating system without any starting values

In summary: No, that is the force at time t. I mean express xmax in terms of x0, v0 and ω.xmax = x0*cos(wt)+v0*sin(wt)*ω
  • #1
RiotRick
42
0

Homework Statement


A mass m1 is located on a platform with mass M. The platfrom is located on springs with total constant k such that it can swing vertically in direction x.

a) Write down the equations of motion assuming mass m1 will always be connected to the platform. Write it as x(t)

b) What's the maximum force on the spring?

c) What's the maximum normal force between m1 and M?

d) What's the Amplitude so that m1 will takeoff from the platform
Thanks in advance

Homework Equations


$$m*\frac{d^2x}{dt^2} = -kx$$

The Attempt at a Solution


a)
I have no information given, so I assume I can't use sin() or cos() or am I overthinking it?
Let m = m1+M
So we have $$m*\frac{d^2x}{dt^2} = -kx$$
And I use the Cauchy–Euler equation $$m*\lambda^2*e^{{\lambda}*t} + k*e^{{\lambda}*t}=0$$
Which leads to: $$\lambda_{1,2} = +-\sqrt{\frac{-k}{m}}$$
Here I have a minus inside the root. I think since no direction is given I can say that I only care about the absolute valueSo I have $$A*e^{\sqrt{k/m}*t}+B*e^{-\sqrt{k/m}*t}$$
There is nothing more that I can do about the constants, because there is nothing given.

For b) and c) I know it has to be on the bottom dead center. d) The acceleration of the mass on the spring has to be greater than gravity
How do I continue from here? My ideas would be to solve it for ##\sqrt{k/m}## or use ##E_{pot} + E_{kin}## but it seems I'm totally off track.
 
Physics news on Phys.org
  • #2
What about the force ##mg##?
 
  • #3
Mister T said:
What about the force ##mg##?
Do I have to add it directly in the "relevant equation" and then treat it like inhomogeneous differential equation 2nd order? or add just afterwards?
 
  • #4
RiotRick said:
Do I have to add it directly in the "relevant equation" and then treat it like inhomogeneous differential equation 2nd order? or add just afterwards?
It depends how you are defining x. Usualy in a vertical spring oscillation one takes x as displacement from equilibrium. This means you are only concerned with the additional spring force over and above that required to oppose g. Thus, your equation is correct.
 
  • #5
RiotRick said:
since no direction is given I can say that I only care about the absolute value
No, the sign matters. Your lambda becomes imaginary, giving terms like e±iωt. You should recognise what that means.
The usual solution avoids this by going straight to the well known solution to ##\ddot x+kx=0##. This is SHM - what form of equation would you expect?
 
  • #6
haruspex said:
No, the sign matters. Your lambda becomes imaginary, giving terms like e±iωt. You should recognise what that means.
The usual solution avoids this by going straight to the well known solution to ##\ddot x+kx=0##. This is SHM - what form of equation would you expect?
The Task says I have to give it as a function x(t). I can solve a to the point where x(t) = ##x_0*cos(wt) + \frac{v_0}{w}*sin(wt)## but how can I use this to solve b,c and d? We can also take Acos()
 
Last edited:
  • #7
For b)
Can I say ##mgx = \frac{1}{2}kx^2## solve it for x and insert it in ##F = -kx## which will give me ##F=2g##
 
  • #8
RiotRick said:
The Task says I have to give it as a function x(t). I can solve a to the point where x(t) = ##x_0*cos(wt) + \frac{v_0}{w}*sin(wt)## but how can I use this to solve b,c and d? We can also take Acos()
If you have quoted the whole question as given to you, there is not enough information. The best you can do is express the answers in terms of the unknowns you have introduced.
 
  • #9
haruspex said:
If you have quoted the whole question as given to you, there is not enough information. The best you can do is express the answers in terms of the unknowns you have introduced.
so for a) the answer is simply ##(m+M)x'' = -kx_{max}##?
 
  • #10
RiotRick said:
so for a) the answer is simply ##(m+M)x'' = -kx_{max}##?
No, xmax is a constant surely?
 
  • #11
haruspex said:
No, xmax is a constant surely?
I'm sorry. I'm talking about b)
 
  • #12
RiotRick said:
I'm sorry. I'm talking about b)
Ok, but express it in terms of the unknowns you introduced in post #6.
 
  • #13
haruspex said:
Ok, but express it in terms of the unknowns you introduced in post #6.
Do you mean ##(m+M)x'' = -k*[x_0*cos(wt)+\frac{v_0}{w}*sin(wt) ##
 
  • #14
RiotRick said:
Do you mean ##(m+M)x'' = -k*[x_0*cos(wt)+\frac{v_0}{w}*sin(wt) ##
No, that is the force at time t. I mean express xmax in terms of x0, v0 and ω.
 
  • #15
haruspex said:
No, that is the force at time t. I mean express xmax in terms of x0, v0 and ω.
The derivative of that function has to be 0 but that's hard to do with that function. I don't see any approach
 
  • #16
RiotRick said:
The derivative of that function has to be 0 but that's hard to do with that function. I don't see any approach
Energy?
 
  • #17
haruspex said:
Energy?
##mgx=1/2*kx^2## putting my function x(t) in will still leave me with cos sin stuff
 
  • #18
RiotRick said:
##mgx=1/2*kx^2## putting my function x(t) in will still leave me with cos sin stuff
I thought we agreed g becomes irrelevant by choice of x=0 at equilibrium?
In terms of your unknowns in post #6, what is the initial energy (excluding GPE)?
 
  • #19
haruspex said:
I thought we agreed g becomes irrelevant by choice of x=0 at equilibrium?
In terms of your unknowns in post #6, what is the initial energy (excluding GPE)?
the initial energy would be ##x_0## and ##v_0##
 
  • #20
RiotRick said:
the initial energy would be ##x_0## and ##v_0##
It would be determined by those, yes, but what is the algebraic expression?
What about when the spring force is maximised?
 
  • #21
haruspex said:
It would be determined by those, yes, but what is the algebraic expression?
What about when the spring force is maximised?
Sorry I don't see it
 
  • #22
RiotRick said:
Sorry I don't see it
This what I had in mind:
In terms of x and v, what is the expression for total mechanical energy (remember, we have neutralised GPE) at any time t?
In terms of the unknowns you introduced in post #6, what is its value at t=0?
In terms of x and v, what is its value when the spring force is at maximum?

However, it was only a suggested approach and I have not checked that it helps. If you prefer, you can wait until I have had a chance to try it myself.

Edit: see next post
 
Last edited:
  • #23
it turns out that using energy leads to the same place, but that place is not as bad as you made out. You just need to use sec2=1+tan2.
Please post your attempt at (b) as far as you can get.
 
  • #24
haruspex said:
it turns out that using energy leads to the same place, but that place is not as bad as you made out. You just need to use sec2=1+tan2.
Please post your attempt at (b) as far as you can get.
I guess you want me to use one of those magic identities. The only one I know is ##a*sin(x) + b*cos(x) = sqrt(a^2 + b^2)*sin(x + t)## so I have ##\sqrt{(x_0)^2+(\frac{v_0}{w})^2}##

So I can read out the Amplitude

For C)

It leads me to ## \frac{F}{m+M} = \sqrt{(x_0)^2+(\frac{v_0}{w})^2}k##
Does this look reasonable?
 
  • #25
RiotRick said:
It leads me to ## \frac{F}{m+M} = \sqrt{(x_0)^2+(\frac{v_0}{w})^2}k##
Does this look reasonable?
Close, but you have a dimensional inconsistency (always something worth checking!)
The LHS is an acceleration but the right is a force.
 
  • #26
haruspex said:
Close, but you have a dimensional inconsistency (always something worth checking!)
The LHS is an acceleration but the right is a force.
Ok step by step.
##\frac{F}{m+M}= x'' = \frac{-kx}{m+M}##
##F_n = \frac{mF}{m+M}= -kx##
##\frac{F}{m+M} = \sqrt{(x_0)^2+(\frac{v_0}{w})^2}\frac{k}{m}##
 
  • #27
RiotRick said:
Ok step by step.
##\frac{F}{m+M}= x'' = \frac{-kx}{m+M}##
##F_n = \frac{mF}{m+M}= -kx##
##\frac{F}{m+M} = \sqrt{(x_0)^2+(\frac{v_0}{w})^2}\frac{k}{m}##
That's got the dimensionality right, but for this part of the question the two masses might as well be one, so you should have M+m both sides, which then cancels.
 
  • Like
Likes RiotRick
  • #28
When I look at this:

RiotRick said:
a) Write down the equations of motion assuming mass m1 will always be connected to the platform. Write it as x(t)

I'm thinking they want two equations. One for ##M## and one for ##m_1##.
 
  • #29
Mister T said:
When I look at this:
I'm thinking they want two equations. One for ##M## and one for ##m_1##.
No, that clause is to cover the fact that if the amplitude exceeds a bound then the objects will separate. This becomes relevant in a later part of the question.
 
  • Like
Likes Mister T
  • #30
Update if anyone runs into the same problem. I don't have a solution but the attempt here is wrong. Right attempt would be ##x''m = -kx + m*g## Which leads to an inhom. diff. equation.

With this I close this thread o7
 
  • #31
RiotRick said:
Update if anyone runs into the same problem. I don't have a solution but the attempt here is wrong. Right attempt would be ##x''m = -kx + m*g## Which leads to an inhom. diff. equation.

With this I close this thread o7
No, as I wrote in post #4 it depends how you define the position x=0. If you define it as being the equilibrium position then that is mg/k below the relaxed spring position. Thus the force in the spring is -k(x-mg/k) = -kx+mg. The net force on the object is thus (-kx+mg)-mg = -kx.
 

1. What is the equation of an oscillating system without any starting values?

The equation of an oscillating system without any starting values is known as the "homogeneous equation" and is written as:
x'' + ω2x = 0
Where x represents the displacement of the system and ω is the natural frequency of the system.

2. How is the equation of an oscillating system without any starting values derived?

The equation of an oscillating system without any starting values is derived from Newton's Second Law of Motion, which states that the sum of all forces acting on a system is equal to the mass of the system times its acceleration. By applying this law to a simple harmonic oscillator, we can derive the above equation.

3. Can the equation of an oscillating system without any starting values be solved analytically?

Yes, the equation of an oscillating system without any starting values can be solved analytically using techniques such as the method of undetermined coefficients or the method of variation of parameters. These techniques involve finding a particular solution and a complementary solution, which together form the general solution to the equation.

4. What is the significance of the natural frequency in the equation of an oscillating system without any starting values?

The natural frequency, denoted by ω, is a measure of how quickly the system will oscillate without any external forces acting on it. It is dependent on the physical properties of the system, such as its mass and stiffness, and can be used to predict the behavior of the system over time.

5. Are there any real-world applications of the equation of an oscillating system without any starting values?

Yes, the equation of an oscillating system without any starting values has many real-world applications, such as in the study of mechanical and electrical systems, sound and light waves, and even biological systems. It is also used in the design of structures, such as bridges and buildings, to ensure they can withstand vibrations and oscillations without collapsing.

Similar threads

  • Introductory Physics Homework Help
Replies
5
Views
535
  • Introductory Physics Homework Help
Replies
2
Views
449
  • Introductory Physics Homework Help
Replies
2
Views
993
  • Introductory Physics Homework Help
Replies
6
Views
993
Replies
31
Views
663
  • Introductory Physics Homework Help
Replies
7
Views
840
  • Introductory Physics Homework Help
Replies
8
Views
334
  • Introductory Physics Homework Help
Replies
13
Views
628
  • Introductory Physics Homework Help
Replies
9
Views
2K
  • Introductory Physics Homework Help
Replies
3
Views
683
Back
Top